5

Consider the subset of odd positive integers defined and constructed as follows by these rules :

A) $1$ is in the set.

B) if $x$ is in the set , then $2x + 1$ is in the set.

C) if $x$ and $y$ are in the set then $xy$ is in the set.

I call them extended Mersenne numbers because rule A and B alone give the Mersenne numbers $2^n - 1$.

And every product of Mersenne numbers must be in the set as well.

So the set or list of extended Mersenne numbers starts like

$$1,3,7,9,15,19,21,27,31,39,43,45,49,55,57,63,79,81,87,91,93,99,...$$

The main question is how dense is this within the odd integers ?

Secondary is there an easy and efficient test to know if an odd integer is in the list ?

I assume this sequence has no name yet but I could be wrong ?


*** speculation ***

I see $22$ odd elements from $1$ till $99$, which is close to the number of odd primes under $99$ (that is $24$) $$ \dfrac11 + \dfrac13 + \dfrac17 + \dfrac19 + \ldots + \ldots+\dfrac{1}{99} = 2.034980..$$

So this (subjectively I admit ) seems to imply the sequence grows quite fast. Therefore I guess maybe an asymptotic of the form

$C* x * \ln(x)^D $

for some constants $C,D$ might be an asymptotic for the counting function.

On the other hand I suspect much sharper asymptotics can be proven.



Background

This comes from abstract algebra where we want a commutative latin square with units and inverses such that "Property A"

$$ x * (y * y^{-1}) = x = (x * y) * y^{-1} $$

holds for all elements $x,y$.

( if this property has a name please tell me )

and for every element $z$

$$ z^2 = 1 $$

( not the main question here, but I wonder how dropping the $z^2 = 1$ condition effects things )

The dimensions of these are then exactly these extended Mersenne numbers + $1$.


edit

corrected forgotten terms and copy lulu's conjecture :

The sequence is equal to https://oeis.org/A197625 ???

( https://math.stackexchange.com/users/252071/lulu )

edit2

Greg Martin found counterexample 219 so lulu's conjecture is false.

Greg also conjectured that $D=0$ or $D$ goes to $0$.

It seems that Greg believes we will get close to linear, but I want to point out that the set is denser than the products of mersenne numbers.

On the other hand numbers like $63 = 3 * 21 = 2*31 + 1$ can be reached in $2$ ways so the rules have overlapping values, which could lower the density.

I would not be surprised if the density is slightly higher than the sum of 2 squares or the density of $a^3 + b^3 + c^3$ which are both less than linear.



It reminds me a bit of collatz, where the rules $2x$ and $(x-1)/3$ generate all integers , or so we believe.

But these rules for the extented Mersenne numbers have no deminishing rule like $(x-1)/3$ ( deminish because that is smaller than $x$) , which is why I believe they might not have linear density.


added

Noticed how powers of $5$ or powers of $17$ are never in the list :

$5$ and $25$ are not in.

So $125,625,…$ are not created By products. Also $125,625,…$ are of the form $4n + 1$, so they did not come from the 2x + 1 rule either.

Similar with $17$ or other primes missing of the form $4n + 1$.

This ofcourse highly influences the density.

Also notice primes of the form $4n+1$ have to come from 2x + 1 themselves , where x must be odd !!

2 x + 1 maps 1 mod 4 to 3 mod 4, and maps 3 mod 4 to 3 mod 4 !!

So primes 1 mod 4 are a key thing here !!


mick
  • 17,886
  • Now posted to overflow, https://mathoverflow.net/questions/440747/density-of-extended-mersenne-numbers without notice to either site. That's an abuse. – Gerry Myerson Feb 14 '23 at 04:38
  • Comments have been moved to chat; please do not continue the discussion here. Before posting a comment below this one, please review the purposes of comments. Comments that do not request clarification or suggest improvements usually belong as an answer, on [meta], or in [chat]. Comments continuing discussion may be removed. – Xander Henderson Feb 14 '23 at 16:09
  • So, mick, have you made any effort to get the Bruck paper from the Albert book? – Gerry Myerson Feb 14 '23 at 22:09
  • Some analysis worth looking at : https://mathoverflow.net/questions/440747/density-of-extended-mersenne-numbers – mick Jun 23 '24 at 19:21
  • An interesting question or reference maybe : Is this the set $S$ in R. H. Bruck, What is a loop?, pp. 59-99 in A. A. Albert, ed., Studies in Modern Algebra, Vol. 2, Mathematical Association of... – mick Jun 23 '24 at 19:24
  • See also : https://math.stackexchange.com/questions/4942552/comparing-two-sets-if-u-is-in-the-set-so-is-2u-1-vs-2u-5-extended-m – mick Jul 06 '24 at 20:06

0 Answers0